Вы находитесь на странице: 1из 5

PHY4604 Fall 2006 Exam 2

PHY 4604 Exam 2


(Total Points = 50)
Problem 1 (20 points)

Problem 1a (2 points): The wave function for an electron in a state with zero angular
momentum: (circle the correct answer)
(a) is zero everywhere (b) is spherically symmetric (c) depends on the angle from the z axis
(d) depends on the angle from the x axis
(e) is spherically symmetric for some shells and depends on the angle from the z axis for others

Problem 1b (2 points): The magnitude of the orbital angular momentum of an electron in an


atom is what multiple of h ? (l = 0, 1, 2, …) (circle the correct answer)
(a) 1 (b) ½ (c) l (l + 1) (d) 2l + 1 (e) l 2

Problem 1c (2 points): An electron is in a quantum state for which the magnitude of the orbital
momentum is 6 2h . How many allowed values of the z-component of the angular momentum
are there? (circle the correct answer)
(a) 4 (b) 5 (c) 7 (d) 8 (e) 9

Problem 1d (2 points): If the wave function ψ is spherically symmetric then the radial
probability density is given by: (circle the correct answer)
(a) 4πr 2ψ (b) | ψ |2 (c) 4πr 2 | ψ |2 (d) 4π | ψ |2 (e) 4πr | ψ |2

Problem 1e (3 points): An electron in an atom is in a state with l = 3 and ml = 2. The angle


r
between L and the z-axis is given by: (circle the correct answer)
(a) 48.2º (b) 60º (c) 30º (d) 35.3º (e) 54.7º

Problem 1f (3 points): SU(2) is the group of 2×2 matrices, U, where:


(a) U ↑U = UU ↑ = 1 (b) U = U ↑ (c) det(U)=1 (d) U −1 = U ↑ (e) U * = U T
(Note: circle all the correct answers)

Problem 1g (3 points): If H is an Hermitian operator then:


(a) H ↑ H = HH ↑ = 1 (b) H = H ↑ (c) <H> ≥ 0
2
(d) <H > ≥ 0 (e) its eigenvalues are real
(Note: circle all the correct answers)
v
Problem 1h (3 points): If L is the orbital angular momentum operator then:
(a) ΔLx ΔLy = 0 (b) ΔLx ΔLy ≥ h2 |< Lz >| (c) ΔLx ΔLy ≥ h2 (d) ΔLz ΔL2 ≥ 0
(Note: circle all the correct answers)

Department of Physics Page 1 of 5


PHY4604 Fall 2006 Exam 2

Problem 2 (30 points): Consider a spin ½ system described by the Hamiltonian:


⎛ε − iε 1 ⎞
H = ⎜⎜ 0 ⎟⎟
⎝ iε 1 ε 0 ⎠
where ε0 and ε1 are real positive constants.
(a) (6 points) Find the energy levels of the system. How many energy levels are there? What is
the ground state energy, E0, and the first excited state energy, E1?

(b) (6 points) What are the (normalized) eigenkets corresponding to the ground state, |E0>, and
the first excited state , |E1>?

(c) (6 points) Now suppose that at t = 0 the system is in the state


⎛1⎞
| ψ 0 >= ⎜⎜ ⎟⎟ .
⎝ 0⎠
If you measure the energy of the state |ψ0>, what are the possible values you might get, and what
is the probability of getting each of them? What is the expectation value of the energy for this
state (i.e. the average energy <E>)?

(d) (6 points) What is <E2> for the state |ψ0> from part (c)? What is the uncertainty in the
energy of the state |ψ0> (i.e. what is ΔE = σE for this state)?

(e) (6 points) If at t = 0 the system is in the state


⎛1⎞
| ψ 0 >= ⎜⎜ ⎟⎟ ,
⎝ 0⎠
what is |ψ(t)> at later time t?

Department of Physics Page 2 of 5


PHY4604 Fall 2006 Exam 2

Problem 3 (25 points): Consider the (one dimensional) wave function at t = 0 given by
ψ ( x ) = Ae − a | x | ,
where A and a are real constants, and where |x| = x for x > 0 and |x| = -x for x < 0.
(a) (4 points) Find the normalization constant A such that ρ(x)
+∞

∫ | ψ ( x) | dx = 1
2

−∞

and sketch the probability density ρ ( x) =| ψ ( x) |2 .

(b) (6 points) Compute <x> , <x2>, and Δx using the position space
wave function ψ(x).

(c) (7 points). Find the momentum space wave function at t = 0, where ρ(px)
+∞
1
φ ( px ) = ∫
2πh − ∞
ψ ( x)e − ixp x / h dx

and verify that it is properly normalized. Sketch the probability


density ρ ( px ) =| φ ( px ) |2 .
px

(d) (6 points). Compute <px>, <px2>, and Δpx using the momentum space wave function φ(px).

(e) (2 points). What is Δx Δpx? Is it consistent with the uncertainty principle?

Department of Physics Page 3 of 5


PHY4604 Fall 2006 Exam 2

Problem 4 (25 points): Suppose we have two vector operators


r r r r
( J1 )op and ( J 2 )op with [( J1 )op , ( J 2 )op ] = 0
and each of the vectors obey the same SU(2) “lie algebra”:
[( J1i )op , ( J1 j )op ] = iε ijk ( J1k )op and [( J 2i )op , ( J 2 j )op ] = iε ijk ( J 2 k )op .
The states |j1m1> are the eigenkets of ( J12 ) op and ( J1z )op and the states |j2m2> are the eigenkets
of ( J 22 ) op and ( J 2 z )op as follows:

( J12 )op | j1m1 >= j1 ( j1 + 1) | j1m1 > ( J 22 )op | j2 m2 >= j2 ( j2 + 1) | j2 m2 >


( J1z )op | j1m1 >= m1 | j1m1 > ( J 2 z )op | j2 m2 >= m2 | j2 m2 >
Also we know that
( J1± )op | j1m1 >= j1 ( j1 + 1) − m1 (m1 ± 1) | j1m1 ± 1 >
( J 2± )op | j2 m2 >= j2 ( j2 + 1) − m2 (m2 ± 1) | j2 m2 ± 1 >
± ±
where ( J1 )op = ( J1x )op ± i ( J1 y )op and ( J 2 )op = ( J 2 x )op ± i ( J 2 y )op . Now consider the vector
sum of the two operators,
r r r
( J )op = ( J1 )op + ( J 2 )op or ( J i )op = ( J i1 )op + ( J i 2 )op for i = 1,2, 3.
(a) (5 points): Show that
r r r r r r r r
( J 2 )op = ( J )op ⋅ ( J )op = ( J1 + J 2 )op ⋅ ( J1 + J 2 )op = ( J12 )op + ( J 22 )op + 2( J1 )op ⋅ ( J 2 )op
= ( J12 )op + ( J 22 )op + ( J1+ )op ( J 2− )op + ( J1− )op ( J 2+ )op + 2( J1z )op ( J 2 z )op

(b) (5 points): Evaluate the following in SU(2).


3×2= 4×3= 5×3=
5×4= 2×3×4=
(c) (15 points): Now consider the case where j1 = 1 and j2 = ½ (i.e. 3 × 2) and define the states
as follows:
| Y11 >1 =| 11 >
|↑>2 =| 12 12 >
| Y10 >1 =| 10 > and
|↓>2 =| 12 − 12 >
| Y1−1 >1 =| 1 − 1 >
Now consider the two superposition states
1 2 2 1
| + >≡ | Y11 >1|↓> 2 + | Y10 >1|↑> 2 and | − >≡ | Y11 >1|↓> 2 − | Y10 >1|↑> 2 .
3 3 3 3
Calculate the following and express your answer in terms of |+> and |->:
(1) ( J1 + J 2 ) | + > (2) ( J1 + J 2 ) | − >
2 2 2 2

+ − − + + − − +
(3) ( J1 J 2 + J1 J 2 + 2 J1z J 2 z ) | + > (4) ( J1 J 2 + J1 J 2 + 2 J1z J 2 z ) | − >
(5) J z | + > (6) J z | − > (7) J | + > (8) J | − >
2 2

Are the states |+> and |-> eigenstates of the J and Jz and if so what are their eigenvalues?

Department of Physics Page 4 of 5


PHY4604 Fall 2006 Exam 2

PHY 4604 Exam 2


Useful Math

1 1 1 sin 2 x
ξ ( p) = 1 + p + p + p + L
2 3 4
∫ sin x cos xdx = 2

ξ ( 2) =
π2 ∫ x sin xdx = sin x − x cos x
6 x sin 2 x
∫ sin xdx = −
2

1 1 1 π2 2 4
1+ 2 + 2 + 2 +L = 2
3 5 7 8 x x sin 2 x cos 2 x
∫ x sin xdx = 4 − 4 − 8
2
π
π
∫ sin mx sin nxdx = 2 δ mn
x3 ⎛ x 2 1 ⎞ x cos 2 x
∫ = − ⎜⎜ − ⎟⎟ sin 2 x −
2 2
0 x sin xdx
π 6 ⎝ 4 8⎠ 4
π
∫ cos mx cos nxdx = 2 δ
0
mn
∫ x cos xdx = cos x + x sin x
x sin 2 x
∫ cos xdx = +
2

2 4
2
x x sin 2 x cos 2 x
∫ x cos xdx = 4 + 4 + 8
2

x3 ⎛ x 2 1 ⎞ x cos 2 x
∫ = + ⎜⎜ − ⎟⎟ sin 2 x +
2 2
x cos xdx
6 ⎝ 4 8⎠ 4

1
∫x e
n −a2 x2
dx = Γ( n2+1 ) , where Γ(x) is the gamma function and Γ(x+1) = xΓ(x).
0
2a n +1
Γ(1) = Γ(2) = 1, Γ(n) = (n-1)! if n is a positive integer, and Γ( 12 ) = π .

1
∫x e
n − ax 2
dx = ( n +1) / 2
Γ( n2+1 )
0
2a
Also,
∞ ∞
Γ(n + 1) cos(bx) π − |b | a
∫ x e dx = ∫x
n − ax
and dx = e for a > 0.
0
a n +1 0
2
+a 2
2a
and
+∞
1 x 1 1 π
∫ (a 2 + x 2 )2 dx = 2a 2 (a 2 + x 2 ) + 2a3 tan ( x / a) ∫ (a
−1
dx = 3
0
2
+x )
2 2
4a
+∞
x2
x 1 x2 π
∫ (a 2 + x 2 )2 dx = − 2(a 2 + x 2 ) + 2a tan ( x / a) ∫0 (a 2 + x 2 )2 dx = 4a
−1

x2 x x x 1
∫ (a 2 + x 2 )4 dx = − 6(a 2 + x 2 )3 + 24a 2 (a 2 + x 2 )2 + 16a 4 (a 2 + x 2 ) + 16a5 tan ( x / a)
−1

x4 a2 x 7x x 1
∫ (a 2 + x 2 )4 dx = 6(a 2 + x 2 )3 − 24(a 2 + x 2 )2 + 16a 2 (a 2 + x 2 ) + 16a3 tan ( x / a)
−1

Department of Physics Page 5 of 5

Вам также может понравиться